You are on page 1of 25

Course 1

May 2001
Answer Key
1
2
3
4
5

E
C
C
E
C

21
22
23
24
25

E
C
A
E
B

6
7
8
9
10

D
B
A
C
E

26
27
28
29
30

B
A
C
D
B

11
12
13
14
15

D
D
E
A
D

31
32
33
34
35

C
E
D
A
C

16
17
18
19
20

A
B
D
B
D

36
37
38
39
40

B
E
D
A
B

A
B
C
D
E

Course 1 Solutions

7
8
8
9
8

May 2001

1.

E
We are given that
5e 5b = p5 = p6 = 6e 6b
It follows that
5 e 6 b
= 5b = e 6 b e5 b = e 6 b+5 b = e b
6 e
5
ln = b
6
5
6
b = ln = ln
6
5

2.

C
First, solve for m such that
500 = 8 + 8 (1.07 ) + ... + 8 (1.07 )

m 1

1 (1.07 )m
(1.07 )m 1
= 8
= 8

1 1.07
0.07

5.375 = (1.07 )

ln (5.375) = m ln (1.07 )

ln (5.375)
= 24.86
ln (1.07 )
We conclude that m = 25 .
m=

3.

C
Observe that
dx
dy
= 2 cos (t / 2 ) and
= 2 cost 2t sin t
dt
dt
Therefore,
dx
2
= 2 cos ( / 4 ) =
= 2
dt t = / 2
2
dy
= 2 cos ( / 2 ) sin ( / 2 ) =
dt t = / 2
It follows that the length of the velocity vector at time t =

( 2)

is given by
2

+ ( ) = 2 + 2 .

Course 1 Solutions

May 2001

4.

E
Let X1, X2, X3, and X4 denote the four independent bids with common distribution
function F. Then if we define Y = max (X1, X2, X3, X4), the distribution function G of Y is
given by
G ( y ) = Pr [Y y ]
= Pr ( X 1 y ) ( X 2 y ) ( X 3 y ) ( X 4 y )
= Pr [ X 1 y ] Pr [ X 2 y ] Pr [ X 3 y ] Pr [ X 4 y ]
= F ( y )

1
3
5
4
y
(1 + sin y ) ,
16
2
2
It then follows that the density function g of Y is given by
g ( y) = G '( y)
=

1
3
(1 + sin y ) ( cos y )
4

3
= cos y (1 + sin y ) ,
4
=

3
5
y
2
2

Finally,
E [Y ] =

5/ 2

5/ 2

3/ 2

yg ( y ) dy

3
ycos y (1 + sin y ) dy
3/ 2 4

Course 1 Solutions

May 2001

5.

C
The domain of X and Y is pictured below. The shaded region is the portion of the domain
over which X<0.2 .

Now observe
1 x

0.2
1 2
6

x
+
y

dydx
=
6
(
)

0 y xy 2 y 0 dx
0 0
0.2
0.2
1
1
2
2
2
= 6 1 x x (1 x ) (1 x ) dx = 6 (1 x ) (1 x ) dx
0
0
2
2

0.2 1
2
3
3
= 6
1 x ) dx = (1 x ) |0.2
(
0 = ( 0.8 ) + 1
0 2
= 0.488

Pr [ X < 0.2] =

6.

0.2

1 x

D
Let
S = Event of a standard policy
F = Event of a preferred policy
U = Event of an ultra-preferred policy
D = Event that a policyholder dies
Then
P [U | D ] =
=

P [ D | U ] P [U ]
P [ D | S ] P [ S ] + P [ D | F ] P [ F ] + P [ D | U ] P [U ]

(0.001)(0.10 )
(0.01)(0.50 ) + (0.005)(0.40 ) + (0.001)(0.10 )

= 0.0141

Course 1 Solutions

May 2001

7.

B
Let us first determine k:
1=
Then

1
0

1
0

kxdxdy =

1k
1 2 1
k
kx |0 dy = dy =
0 2
0 2
2
1

k =2
1 1

E [ X ] = 2 x 2 dydx = 2 x 2 dx =
0

0 0

1 1

E [Y ] = y 2 x dxdy = ydy =
0

0 0

E [ XY ] =
=

1
0

1
0

2x 2 ydxdy =

2 3 1 2
x |0 =
3
3
1 2 1 1
y |0 =
2
2

12
2 3 1
x y |0 dy = ydy
0 3
0 3
1

2 2 1 2 1
y |0 = =
6
6 3

1 2 1 1 1
Cov [ X , Y ] = E [ XY ] E [ X ] E [Y ] = = = 0
3 3 2 3 3
(Alternative Solution)
Define g(x) = kx and h(y) = 1 . Then
f(x,y) = g(x)h(x)
In other words, f(x,y) can be written as the product of a function of x alone and a function
of y alone. It follows that X and Y are independent. Therefore, Cov[X, Y] = 0 .
8.

A
By the chain rule,
1 1
1 1
dp d
dx
dy
= 100 xy = 50 x 2 y 2
+ 50 x 2 y 2
dt dt
dt
dt
At the time t0 in question, we are told that
dx
dy
1
x=2 ,
= 1 , y = 3 , and
=
dt
dt
2
Therefore,
dp
3
2
1
= 50
(1) + 50 = 40.8
dt t =t0
2
3 2

Course 1 Solutions

May 2001

9.

C
The Venn diagram below summarizes the unconditional probabilities described in the
problem.

In addition, we are told that


P[A B C]
1
x
= P [ A B C | A B] =
=
3
P [ A B]
x + 0.12
It follows that
1
1
x = ( x + 0.12 ) = x + 0.04
3
3
2
x = 0.04
3
x = 0.06
Now we want to find
c
P ( A B C )
c

P ( A B C ) | Ac =
c

P A
1 P[A B C]
=
1 P [ A]

Course 1 Solutions

1 3 ( 0.10 ) 3 (0.12 ) 0.06


1 0.10 2 ( 0.12 ) 0.06

0.28
= 0.467
0.60

May 2001

10.

E
Let
W = event that wife survives at least 10 years
H = event that husband survives at least 10 years
B = benefit paid
P = profit from selling policies
Then
and

Pr [ H ] = P [ H W ] + Pr H W c = 0.96 + 0.01 = 0.97


Pr [W | H ] =

Pr [W H ]

Pr W | H =
c

Pr [ H ]

0.96
= 0.9897
0.97

Pr H W c
Pr [ H ]

0.01
= 0.0103
0.97

It follows that
E [ P ] = E [1000 B ]

= 1000 E [ B ]

= 1000 ( 0 ) Pr [W | H ] + (10, 000 ) Pr W c | H


= 1000 10, 000 ( 0.0103)
= 1000 103
= 897

Course 1 Solutions

May 2001

11.

D
Observe that x and y follow the constraint equation
x + y = 160, 000
x = 160, 000 y where 0 y 160, 000
Now this constraint equation can be used to express policy sales g(x, y) as a function f(y)
of marketing y alone:
f ( y ) = g (160, 000 y, y ) = 0.001(160, 000 y ) y 3/ 4
We can then compute f '(y) as follows:
3
3/ 4 3/ 4
1/ 4
1

f ' ( y ) = (160, 000 y )


y + (160, 000 y ) y 1/ 4 /1000
4
4

1
3/ 4
=
(160, 000 y ) y 1/ 4 y 3 (160, 000 y )
4000
1
3/ 4
=
(160, 000 y ) y 1/ 4 ( 4 y 480, 000 )
4000
1
3/ 4
=
(160, 000 y ) y 1/ 4 (120, 000 y ) , 0 y 160, 000
1000
Note that
f ' ( y ) > 0 for 0 y < 120, 000 ,
1/ 4

f '( y ) = 0

for y = 120, 000 , and

f ' ( y ) < 0 for 120, 000 < y < 160, 000


We conclude that sales are maximized when y = 120,000 . Therefore,
1/ 4
3/ 4
f (120, 000 ) = 0.001(160, 000 120, 000 ) (120, 000 ) = 91.2 maximizes f .

Course 1 Solutions

May 2001

11.

Alternate solution using Lagrange multipliers:


Solve:
x + y 160, 000 = 0
1

x 4y 4

= ( x + y 160, 000 )
x 1000
x
x 4y 4

= ( x + y 160, 000 )
y 1000
y
From last two equations:
3
3
1
x 4y 4 =
4000
1
1
3
x 4y 4 =
4000
Eliminating :
1

3x 4 y 4 = x 4 y
3x = y
Using first equation:
4 x = 160, 000
x = 40, 000
y = 120, 000

Extreme value (which must be a maximum) is


12.

( 40, 000 ) 4 (120, 000 ) 4


1000

= 91.2

D
First note
P [ A B ] = P [ A] + P [ B ] P [ A B ]

P [ A B '] = P [ A] + P [ B '] P [ A B ']


Then add these two equations to get
P [ A B ] + P [ A B '] = 2 P [ A] + ( P [ B ] + P [ B ']) ( P [ A B ] + P [ A B '])
0.7 + 0.9 = 2 P [ A] + 1 P ( A B ) ( A B ')

1.6 = 2 P [ A] + 1 P [ A]
P [ A] = 0.6

Course 1 Solutions

May 2001

13.

E
Let
X = number of group 1 participants that complete the study.
Y = number of group 2 participants that complete the study.
Now we are given that X and Y are independent.
Therefore,
P ( X 9 ) (Y < 9 ) ( X < 9 ) (Y 9 )

= P ( X 9 ) (Y < 9 ) + P ( X < 9 ) (Y 9 )
= 2 P ( X 9 ) (Y < 9 )

(due to symmetry)

= 2 P [ X 9] P [Y < 9]

= 2 P [ X 9] P [ X < 9]

(again due to symmetry)

= 2 P [ X 9](1 P [ X 9])
9
10
9
10
10
1 ( 10

= 2 ( 10
0.2 )( 0.8 ) + ( 10
(
)
10 ) ( 0.8 )
9 ) ( 0.2 )( 0.8 ) ( 10 ) ( 0.8 )
9

= 2 [0.376][1 0.376] = 0.469

14.

A
Let f1(x) denote the marginal density function of X. Then
f1 ( x ) =

x +1

Consequently,

f ( y| x ) =

2 xdy = 2 xy |xx +1 = 2 x ( x + 1 x ) = 2 x

0 < x <1

f ( x, y ) 1 if: x < y < x + 1


=
f1 ( x ) 0 otherwise

1 2 x +1 1
1
1
1 1
1
2
y |x = ( x + 1) x 2 = x 2 + x + x 2 = x +
2
2
2
2
2 2
2
x +1
1
1
1
3
E Y 2 | X = y 2 dy = y 3 |xx +1 = ( x + 1) x 3
x
3
3
3
1
1 1
1
= x3 + x 2 + x + x3 = x 2 + x +
3
3 3
3
E [Y | X ] =

x +1

ydy =

2
1
1
Var [Y | X ] = E Y 2 | X {E [Y | X ]} = x 2 + x + x +
3
2
1
1 1
= x2 + x + x2 x =
3
4 12

Course 1 Solutions

10

May 2001

15.

D
At the point (0, 5),

0 = 2t 2 + t 1 = ( 2t 1)(t + 1)

5 = t 2 3t + 1
1
The first equation says t = or t = 1 and the second says t = 1 .
2
The slope of the tangent line to C at (0, 5) is then
dy
2t 3
dy
dx
=
=
dx ( x , y )=(0,5) dt t =1 dt t =1 4t + 1 t =1
and

5
= 2 ( 1) 3 4 ( 1) + 1 =
3
16.

A
We are given that

0
for 0 x 10

0.02 ( x 10 )
T (x) =
for 10 < x 20
x

0.04 ( x 20 ) + 0.02 (10 )


for x > 20

0
for 0 x 10

T ( x) = 0.02
for 10 < x 20
5
x

0.04 5 x for x > 20

1
Therefore, T ' ( x ) = 2
5x
3
5 x 2

0
for 0 < x < 10

2
T " ( x ) = 3 for 10 < x < 20
5x
6
5 x 3 for x > 20

for 0 < x < 10


for 10 < x < 20
for x > 20

and

We can infer the following about T(x):


i) T ( x ) = 0 for 0 < x 10

ii) T ( x ) is strictly increasing for 10 < x < 20

and x > 20 since T ' ( x ) > 0 on both of these intervals.

iii) T ( x ) is concave down for 10 < x < 20

and x > 20 since T " ( x ) < 0 on both of these intervals.


It follows that (A) is the only graph that satisfies conditions (i)-(iii).

Course 1 Solutions

11

May 2001

17.

B
Let Y denote the claim payment made by the insurance company.
Then
with probability 0.94
0

Y = Max ( 0, x 1) with probability 0.04


14
with probability 0.02

and
E [Y ] = ( 0.94 )(0 ) + ( 0.04 )( 0.5003)

15

( x 1) e x / 2 dx + (0.02 )(14 )

15
15
= ( 0.020012 ) xe x / 2 dx e x / 2 dx + 0.28
1
1

15
15
= 0.28 + ( 0.020012 ) 2 xe x / 2 |115 +2 e x / 2 dx e x / 2 dx

1
1
15
= 0.28 + ( 0.020012 ) 30e7.5 + 2e0.5 + e x / 2 dx
1

= 0.28 + ( 0.020012 ) 30e7.5 + 2e0.5 2e x / 2 |115

= 0.28 + ( 0.020012 ) ( 30e 7.5 + 2e 0.5 2e 7.5 + 2e 0.5 )


= 0.28 + ( 0.020012 ) ( 32e 7.5 + 4e 0.5 )
= 0.28 + ( 0.020012 )( 2.408 )
= 0.328

(in thousands)

It follows that the expected claim payment is 328 .


18.

D
By the chain rule,
f
u
v
= veuv
+ ueuv
= veuv 2 x + ueuv 2 y
y
y
y
Therefore,
f
4 5
4 5
= 5e( )( ) ( 2 )( 2 ) + 4e( )( ) ( 2 )(1) = 28e20
y ( x , y )=( 2,1)

Course 1 Solutions

12

May 2001

19.

B
Let X1,, Xn denote the life spans of the n light bulbs purchased. Since these random
variables are independent and normally distributed with mean 3 and variance 1, the
random variable S = X1 + + Xn is also normally distributed with mean
= 3n
and standard deviation
= n
Now we want to choose the smallest value for n such that
S 3n 40 3n
0.9772 Pr [ S > 40] = Pr
>

n
n
This implies that n should satisfy the following inequality:
40 3n
2
n
To find such an n, lets solve the corresponding equation for n:
40 3n
2 =
n
2 n = 40 3n
3n 2 n 40 = 0

(3

n + 10

)(

n 4 =0
n =4
n = 16

20.

D
The density function of T is
1
f (t ) = e t / 3 , 0 < t <
3
Therefore,
E [ X ] = E max (T , 2 )
=

t
2 t / 3
e dt + e t / 3 dt
2 3
3

= 2e t / 3 |02 te t / 3 |2 + e t / 3 dt
2

= 2e

2 / 3

+ 2 + 2e

2 / 3

3e t / 3 |2

= 2 + 3e 2 / 3

Course 1 Solutions

13

May 2001

21.

E
The differential equation that we are given is separable. As a result, the general solution
is given by
1
Q ( N Q ) dQ = dt = t + C
where C is a constant. Now in order to calculate the integral on the lefthand side of this
equation, we first need to determine the partial fractions of the integrand. In other words,
we need to find constants A and B such that
1
A
B
= +
Q (N Q) Q N Q
1 = A ( N Q ) + BQ

1 = AN + ( B A) Q
It follows that
AN = 1
BA=0
1
B= A=
N
1
1
1
so
=
+
and
Q ( N Q ) NQ N ( N Q )
1

Q ( N Q ) dQ = N Q dQ + N N Q dQ = N 1nQ N 1n ( N Q ) + K = N 1n N Q + K
where K is a constant. Consequently,
1 Q
ln
+ K =t +C
N N Q
1/ N

N Q

e K = et eC

1/ N

Q
t CK

=ee
N Q
Q
N CK
= e Nt e ( )
N Q
Q = ae Nt ( N Q ) = aNe Nt ae Nt Q where a = e

(1 + ae ) Q = aNe
Nt

Q (t ) =

Course 1 Solutions

N (C K )

is a constant

Nt

aNe Nt
1 + ae Nt

14

May 2001

22.

C
Let X denote the waiting time for a first claim from a good driver, and let Y denote the
waiting time for a first claim from a bad driver. The problem statement implies that the
respective distribution functions for X and Y are
F ( x ) = 1 e x / 6 , x > 0
and
G ( y ) = 1 e y / 3 , y > 0
Therefore,
Pr ( X 3) (Y 2 ) = Pr [ X 3] Pr [Y 2]
= F (3) G ( 2 )

= (1 e1/ 2 )(1 e2 / 3 )
= 1 e2 / 3 e1/ 2 + e7 / 6
23.

A
Let
C = Event that shipment came from Company X
I1 = Event that one of the vaccine vials tested is ineffective
P [ I1 | C ] P [C ]
Then by Bayes Formula, P [C | I1 ] =
P [ I1 | C ] P [C ] + P I1 | C c P C c
Now
1
P [C ] =
5
1 4
P C c = 1 P [C ] = 1 =
5 5
P [ I1 | C ] = ( 130 ) ( 0.10 )( 0.90 ) = 0.141
29

P I1 | C c = ( 130 ) ( 0.02 )(0.98 ) = 0.334


29

Therefore,

P [C | I1 ] =

Course 1 Solutions

(0.141)(1/ 5)
= 0.096
(0.141)(1/ 5) + (0.334 )( 4 / 5 )

15

May 2001

24.

E
The domain of s and t is pictured below.

Note that the shaded region is the portion of the domain of s and t over which the device
fails sometime during the first half hour. Therefore,
1/ 2 1
1 1/ 2

1
1
Pr S T = f ( s, t ) dsdt + f ( s, t ) dsdt
0 0
2
2 0 1/ 2

(where the first integral covers A and the second integral covers B).
25.

B
Note that V, S and r are all functions of time t. Therefore,
dV
dr
= 4 r 2
dt
dt
and
dS
dr
= 8 r
dt
dt
We are given that
dV
6
= 60 when r = = 3 .
dt
2
It follows that
2 dr
60 = 4 (3)
dt
dr
5
=
dt 3
dS
5
= 8 (3) = 40
dt
3

Course 1 Solutions

16

May 2001

26.

B
Let
u be annual claims,
v be annual premiums,
g(u, v) be the joint density function of U and V,
f(x) be the density function of X, and
F(x) be the distribution function of X.
Then since U and V are independent,
1
1
g (u , v ) = ( e u ) e v / 2 = e u e v / 2 , 0 < u < , 0 < v <
2
2
and
u

F ( x ) = Pr [ X x ] = Pr x = Pr [U Vx ]
v

vx
vx 1
= g (u , v )dudv =
e u e v / 2 dudv
0 0
0 0 2

1
1
1

= e u e v / 2 |vx0 dv = e vx e v / 2 + e v / 2 dv
0
0
2
2
2

1 v x +1/ 2) 1 v / 2
= e (
+ e dv
0
2
2

+1/ 2
e v( x ) e v / 2
=
2x +1
0
1
=
+1
2x +1

Finally,

f ( x) = F '( x) =

Course 1 Solutions

( 2 x + 1)

17

May 2001

27.

A
First, observe that the distribution function of X is given by
x 3
1
1
F ( x ) = 4 dt = 3 |1x = 1 3 , x > 1
1 t
t
x
Next, let X1, X2, and X3 denote the three claims made that have this distribution. Then if
Y denotes the largest of these three claims, it follows that the distribution function of Y is
given by
G ( y ) = Pr [ X 1 y ] Pr [ X 2 y ] Pr [ X 3 y ]
3

1
= 1 3
, y >1
y
while the density function of Y is given by
2

1 3 9
1
g ( y ) = G ' ( y ) = 3 1 3 4 = 4 1 3
y y y y
Therefore,
E [Y ] =

y >1

9
9
1
2
1
1 3 dy = 3 1 3 + 6 dy
3
1 y
y y
y
y

9 18 9
9
18
9
= 3 6 + 9 dy = 2 + 5 8
1
y
y
y
2 y 5 y 8 y 1
1 2 1
= 9 + = 2.025 (in thousands)
2 5 8

Course 1 Solutions

18

May 2001

28.

C
Since f (t ) > 0

and f ' (t ) < 0

(i ) f (t0 ) > f (t1 )

for t 0 , the following inequalities hold:

if

0 t0 < t1

(ii ) f ( k ) < k 1 f (t )dt

if k 1

k +1

(iii ) f ( k ) > k

f (t )dt

k 0

if

Applying these inequalities, we see that


f ( 0 ) + f (1) +

20

= f (0 ) +

20

20

f (t ) dt > f ( 0 ) + f (t ) dt +
1

> f (1) +

20

20

f (t ) dt = f (t ) dt +

f (t ) dt > f (t ) dt +
0

20

f (t ) dt

f (t ) dt

f (t ) dt

f (t ) dt = f (1) + k =1
19

k +1

20

f (t ) dt

> f (1) + k =1 f ( k + 1) = f (1) + k =2 f ( k )


19

>

f ( k ) > k =1
k =1
20

20

20

20

k +1

21

f (t ) dt = f (t )dt
1

f (t ) dt

We conclude that f (1) +

20

f (t )dt produces the smallest number that exceeds

N = k =1 f ( k ) .
20

Course 1 Solutions

19

May 2001

28.

Note a more heuristic approach to the result that (E) > (B) > (A) > (C) >

20

f ( k ) > (D)
k =1

can be obtained from diagrams of the following sort:

20

20

gives ( E ) = f (0 ) + f (1) + f (t ) dt > f ( 0 ) + f (t ) dt = ( B )


and

20

20

k =1

gives f (1) + f (t ) dt > f (t )


29.

D
Let
X = number of low-risk drivers insured
Y = number of moderate-risk drivers insured
Z = number of high-risk drivers insured
f(x, y, z) = probability function of X, Y, and Z
Then f is a trinomial probability function, so
Pr [ z x + 2] = f ( 0, 0, 4 ) + f (1, 0,3 ) + f ( 0,1,3) + f ( 0, 2, 2 )
= ( 0.20 ) + 4 ( 0.50 )( 0.20 ) + 4 ( 0.30 )( 0.20 ) +
4

4!
2
2
( 0.30 ) ( 0.20 )
2!2!

= 0.0488

Course 1 Solutions

20

May 2001

30.

B
Let
x = number of ice cream cones sold
p(x) = price of x ice cream cones
C(x) = cost of selling x ice cream cones
R(x) = revenue from selling x ice cream cones
P(x) = profit from selling x ice cream cones
We are told that p(x) satisfies the following relationship:
p (x) 2
x = 500 5
= 500 500 p ( x ) + 1000 = 1500 500 p ( x )
0.01
500 p ( x ) = 1500 x
p ( x) = 3

x
500

Therefore,
R ( x ) = xp ( x ) = 3 x
C ( x ) = 0.10 x + 75

x2
500

x2
x2
0.10 x 75 = 2.9 x
75
500
500
Now, since P(x) is quadratic, it is clear that P(x) will be maximized for x such that
x
0 = P ' ( x ) = 2.9
250
x
= 2.9
250
x = 725
The profit maximizing price is thus
725
p ( 725 ) = 3
= 1.55
500
P ( x ) = R ( x ) C ( x ) = 3x

Course 1 Solutions

21

May 2001

31.

C
A Venn diagram for this situation looks like:

We want to find w = 1 ( x + y + z )
1
1
5
We have x + y = , x + z = , y + z =
4
3
12
Adding these three equations gives
1 1 5
(x + y) + (x + z) + ( y + z) = + +
4 3 12
2(x + y + z) = 1
x+ y+ z =

1
2

1 1
=
2 2
Alternatively the three equations can be solved to give x = 1/12, y = 1/6, z =1/4
1 1 1 1
again leading to w = 1 + + =
12 6 4 2
w = 1 (x + y + z) = 1

32.

E
Let X and Y denote the times that the two backup generators can operate. Now the
variance of an exponential random variable with mean is 2 . Therefore,
Var [ X ] = Var [Y ] = 102 = 100
Then assuming that X and Y are independent, we see
Var [ X+Y ] = Var [X ] + Var [Y ] = 100 + 100 = 200

Course 1 Solutions

22

May 2001

33.

D
Let
IA = Event that Company A makes a claim
IB = Event that Company B makes a claim
XA = Expense paid to Company A if claims are made
XB = Expense paid to Company B if claims are made
Then we want to find
Pr I AC I B ( I A I B ) ( X A < X B )

= Pr I AC I B + Pr ( I A I B ) ( X A < X B )

= Pr I AC Pr [ I B ] + Pr [ I A ] Pr [ I B ] Pr [ X A < X B ]

(independence)

= ( 0.60 )( 0.30 ) + ( 0.40 )( 0.30 ) Pr [ X B X A 0]


= 0.18 + 0.12 Pr [ X B X A 0]

Now X B X A is a linear combination of independent normal random variables.


Therefore, X B X A is also a normal random variable with mean
M = E [ X B X A ] = E [ X B ] E [ X A ] = 9, 000 10, 000 = 1, 000

and standard deviation = Var ( X B ) + Var ( X A ) =

( 2000 ) + ( 2000 )
2

= 2000 2

It follows that
1000

Pr [ X B X A 0 ] = Pr Z
2000 2

= Pr Z
2 2

= 1 Pr Z <
2 2

(Z is standard normal)

= 1 Pr [ Z < 0.354]

Finally,

= 1 0.638 = 0.362

Pr I AC I B ( I A I B ) ( X A < X B ) = 0.18 + ( 0.12 )( 0.362 )


= 0.223
34.

A
d
[ x 1] .
dx
(Note graph (D) can be eliminated because both curves have non-zero slopes where the
other crosses the x-axis.)
The graph (A) contains the curves y = x 1 and y = 1 =

Course 1 Solutions

23

May 2001

35.

C
if 0 X 4
if 4 < X 5

X
Note Y =
4
Therefore,

54
1
1
4
xdx + dx = x 2 |04 + x|54
0 5
4 5
10
5
16 20 16 8 4 12
= +
= + =
10 5 5 5 5 5
41
5 16
1
16
E Y 2 = x 2 dx +
dx = x 3 |04 + x|54
0 5
4 5
15
5
64 80 64 64 16 64 48 112
=
+
=
+ =
+
=
15 5
5 15 5 15 15 15

E [Y ] =

Var [Y ] = E Y 2 ( E [Y ]) =
2

36.

112 12
= 1.71
15 5

B
Let T denote the total concentration of pollutants over the town. Then due to symmetry,
T = 4

22,500 (8 x 2 y 2 ) dxdy
2

= ( 4 )( 7500 ) 24 x x3 3 xy 2 dy
0
0
= 30, 000 ( 48 8 6 y 2 ) dy
2

= 30, 000 ( 40 6 y 2 )dy


2

= 30, 000 40 y 2 y 3 = 30, 000 (80 16 )


0

= 30, 000 ( 64 ) = 1,920, 000


And since the town covers 16 square miles, it follows that the average pollution
concentration A is A = T /16 = 1,920, 000 /16 = 120,000
37.

E
Observe that the bus driver collect 21x50 = 1050 for the 21 tickets he sells. However, he
may be required to refund 100 to one passenger if all 21 ticket holders show up. Since
passengers show up or do not show up independently of one another, the probability that
all 21 passengers will show up is (1 0.02 ) = (0.98 ) = 0.65 . Therefore, the tour
21

21

operators expected revenue is 1050 (100 )(0.65 ) = 985 .

Course 1 Solutions

24

May 2001

38.

D
From f , observe that
4 x + c1 for 0 < x < 10

f ( x ) = kx + c2 for 10 < x < 30


3 x + c for x > 30
3

As a result, 200 = f(50) = 3(50) + c3 = 150 + c3 implies c3 = 50


And 0 = f ( 0 ) = 4 ( 0 ) + c1 = c1 ,
Then due to the continuity requirement,
10k + c2 = f (10 ) = 4 (10 ) + c1 = 40 + 0 = 40, and

30k + c2 = f (30 ) = 3 (30 ) + c3 = 90 + 50 = 140


Solving these last two equations simultaneously, we see that
20k = 100 or k = 5 .

39.

A
Let F denote the distribution function of f. Then
x

F ( x ) = Pr [ X x ] = 3t 4 dt = t 3 = 1 x 3
1

Using this result, we see

Pr [ X < 2| X 1.5] =

Pr ( X < 2 ) ( X 1.5 )
Pr [ X 1.5]

Pr [ X < 2 ] Pr [ X 1.5]
Pr [ X 1.5]

F ( 2 ) F (1.5) (1.5 ) ( 2 )
=
=
3
1 F (1.5 )
(1.5 )
3

40.

3
= 1
4

= 0.578

B
Let
H = event that a death is due to heart disease
F = event that at least one parent suffered from heart disease
Then based on the medical records,
210 102 108
P H F c =
=
937
937
937

312
625
P F c =
=
937
937
c
P H F 108 625 108
and P H | F c =
=
=
= 0.173
937 937 625
P F c

Course 1 Solutions

25

May 2001

You might also like